Homc PDF
Homc PDF
Question 2. How many pairs of positive integers (x, y) are there, those satisfy
the identity
2x − y 2 = 1?
(A): 1 (B): 2 (C): 3 (D): 4 (E): None of the above.
Answer. The choice is (A).
Question 4. Put
2m p2 + 27 = q 3 , and p is a prime.
2m p2 = (q − 3)(q 2 + 3q + 9).
1
Remark that q 2 + 3q + 9 is always odd. There are two cases:
Case 1. q = 2m p + 3. We have
which is impossible.
Case 2. q = 2m + 3. We have
which implies
p2 = 22m + 9 × 2m + 27.
If m ≥ 3, then 22m + 9 × 2m + 27 ≡ 3 (mod 8), but p2 ≡ 1 (mod 8). We deduce
m ≤ 3. By simple computation we find m = 1, p = 7, q = 5.
Q = 22017 + 20172 .
Solution. We have
201
22017 = 27 × 210 = 128 × 1024201
≡ 128 × (−1)201 = −128 ≡ 22 (mod 25);
20172 ≡ 14 (mod 25).
It follows P ≡ 11 (mod 25), by which two last digits of P are in the set 11, 36, 61, 86 .
In other side, P ≡ 1 (mod 4). This implis P ≡ 61 (mod 100). Thus, the number
61 subjects to the question.
By 2y 3 − 4 − 6y = 2 − z, we have
2
Similarly, by 3z 3 − 3 − 3 − 9z = 2 − x we have
which is impossible.
3
We cover S by A, B, C as Figure 1. We see that A, B, C will have common
parts, mutually. Suppose
X = B ∩ C; Y = A ∩ C; Z = A ∩ B.
It follows
X + Y ≤ C; Y + Z ≤ A; Z + X ≤ B.
We deduce A, B, C cover a part of area:
1
A+B+C −X −Y −Z ≥A+B+C − (X + Y ) + (Y + Z) + (Z + X)
2
1 D+E
≥ (A + B + C) = 1 − ≥ 1 − D.
2 2
Thus, D can cover the remained part of S.
Question 10. Find all non-negative integers a, b, c such that the roots of
equations:
x2 − 2ax + b = 0; (1)
x2 − 2bx + c = 0; (2)
x2 − 2cx + a = 0 (3)
a2 − b = p 2 ; b2 − c = q 2 ; c2 − a = r 2 .
Question 11. Let S denote a square of the side-length 7, and let eight squares
of the side-length 3 be given. Show that S can be covered by those eight small
squares.
Solution.
Figure 2 is a solution.
Question 12. Does there exist a sequence of 2017 consecutive integers which
contains exactly 17 primes?
4
Figure 2: For Question 11
Solution. It is easy to see that there are more than 17 primes in the sequence
of numbers 1, 2, 3, 4, . . . , 2017. Precisely, there are 306 primes in that sequence.
Remark that if the sequence
k + 1, k + 2, . . . , k + 2017
k, k + 1, . . . , k + 2016,
then the numbers of primes in the latter and former sequences are either equal,
more or less by 1. In what follows, we say the such change a shift back with 1
step. First moment, we consider the sequence of 2017 consecutive integers:
which contain no prime. After 2018!+1 times shifts back, we obtain the sequence
1, 2, 3, 4, . . . , 2017.
The last sequence has 306 primes, while the first sequence has no prime. Remind-
ing the above remark we conclude that there is a moment in which the sequence
contains exactly 17 primes.
Question 13. Let a, b, c be the side-lengths of triangle ABC with a+b+c = 12.
Determine the smallest value of
a 4b 9c
M= + + .
b+c−a c+a−b a+b−c
5
Solution. Put
b+c−a c+a−b a+b−c
x := , y := , z := .
2 2 2
Then x, y, z > 0, and
a+b+c
x+y+z = = 6, a = y + z, b = z + x, c = x + y.
2
We have
y + z 4(z + x) 9(x + y) 1 y 4x z 9x 4z 9y
M= + + = + + + + +
2x 2y 2z 2 x y x z y z
r r r !
1 y 4x z 9x 4z 9y
≥ 2 . +2 . +2 . = 11.
2 x y x z x z
Question 14. Given trapezoid ABCD with bases AB k CD (AB < CD).
Let O be the intersection of AC and BD. Two straight lines from D and C are
perpendicular to AC and BD intersect at E, i.e. CE ⊥ BD and DE ⊥ AC.
By analogy, AF ⊥ BD and BF ⊥ AC. Are three points E, O, F located on the
same line?
Solution. Since E is the orthocenter of triangle ODC, and F is the orthocenter
of triangle OAB we see that OE is perpendicular to CD, and OF is perpendicular
to AB. As AB is parallel to CD, we conclude that E, O, F are straightly lined.
Question 15. Show that an arbitrary quadrilateral can be divided into nine
isosceles triangles.
Solution. Figures 3, 4, and 5 shows some solution.
6
Figure 3: For Question 15
7
Answer. The solution is (A).
Question 3. The number of real triples (x, y, z) that satisfy the equation
x4 + 4y 4 + z 4 + 4 = 8xyz
is
(A): 0; (B): 1; (C): 2; (D): 8; (E): None of the above.
Answer. The solution is (E).
Question 4. Let a, b, c be three distinct positive numbers. Consider the quadratic
polynomial
c(x − a)(x − b) a(x − b)(x − c) b(x − c)(x − a)
P (x) = + + + 1.
(c − a)(c − b) (a − b)(a − c) (b − c)(b − a)
The value of P (2017) is
(A): 2015 (B): 2016 (C): 2017 (D): 2018 (E): None of the above.
Answer. The solution is (D).
Question 5. Write 2017 following numbers on the blackboard:
1008 1007 1 1 2 1007 1008
− ,− ,...,− , 0, , ,..., , .
1008 1008 1008 1008 1008 1008 1008
One processes some steps as: erase two arbitrary numbers x, y on the blackboard
and then write on it the number x + 7xy + y. After 2016 steps, there is only one
number. The last one on the blackboard is
1 1 144
(A): − (B): 0 (C): (D): − (E): None of the above.
1008 1008 1008
Answer. The solution is (D).
Question 6. Find all pairs of integers a, b such that the following system of
equations has a unique integral solution (x, y, z)
(
x+y =a−1
x(y + 1) − z 2 = b.
8
This implies the system has a unique solution
a a
(x, y + 1, z) = , ,0 .
2 2
.
Question 7. Let two positive integers x, y satisfy the condition x2 + y 2 .. 44.
Determine the smallest value of T = x3 + y 3 .
.
Solution. Note that 44 = 4 × 11 It follows x2 + y 2 .. 11. We shall prove that
. .
x .. 11 and y .. 11. Indeed, if x and y are not divisible by 11 then by the Fermat’s
little theorem, we have
x10 + y 10 ≡ 2 (mod 11). (1)
.
On the other hand, since x2 + y 2 .. 11 then x2 + y 2 ≡ 0 (mod 11). It follows
x10 + y 10 ≡ 0 (mod 11),
. . .
which is not possible by (1). Hence, x .. 11 or y .. 11 and that follow x .. 11 and
. . . .
y .. 11 simultaneously (if x .. 11 then from x2 + y 2 .. 11. It follows y 2 .. 11 and
. .
then y .. 11). In other side, we have x2 + y 2 .. 4 and x2 ≡ 0, 1 (mod 4), y 2 ≡ 0, 1
. .
(mod 4). We then have x2 ≡ 0 (mod 4), y 2 ≡ 0 (mod 4). It follows x .. 2, y .. 2.
. .
Since (2, 11) = 1, x .. (22) and y .. (22). Thus, min A = (22)3 + (22)3 = 21296.
Question 8. Let a, b, c be the side-lengths of triangle ABC with a + b + c = 12.
Determine the smallest value of
a 4b 9c
M= + + .
b+c−a c+a−b a+b−c
b+c−a c+a−b a+b−c
Solution. Let x = ,y = ,z = then x, y, z > 0 and
2 2 2
a+b+c
x+y+z = = 6, a = y + z, b = z + x, c = x + y. We have
2
y + z 4(z + x) 9(x + y) 1 y 4x z 9x 4z 9y
M= + + = + + + + +
2x 2y 2z 2 x y x z y z
r r r !
1 y 4x z 9x 4z 9y
≥ 2 . +2 . +2 . = 11.
2 x y x z y z
The equality yields if and only if
y 4x
=
x y
z 9x
=
x z
4z 9y
= .
y z
9
Equivelently,
y = 2x
z = 3x
2z = 3y.
Question 9. Cut off a square carton by a straight line into two pieces, then cut
one of two pieces into two small pieces by a straight line, ect. By cutting 2017
times we obtain 2018 pieces. We write number 2 in every triangle, number 1 in
every quadrilateral, and 0 in the polygons. Is the sum of all inserted numbers
always greater than 2017?
Question 10. Consider all words constituted by eight letters from {C, H, M, O}.
We arrange the words in an alphabet sequence. Precisely, the first word is CC-
CCCCCC, the second one is CCCCCCCH, the third is CCCCCCCM, the fourth
one is CCCCCCCO,. . . , and the last word is OOOOOOOO.
a) Determine the 2017th word of the sequence?
b) What is the position of the word HOMCHOMC in the sequence?
{13201320}4 = 47 + 3 × 46 + 2 × 45 + 0 × 44 + 1 × 43 + 3 × 42 + 2 × 4 + 0.
A simple computation gives {13201320}4 = 30840. Thus, the word HOM CHOM C
is 30840th in the sequence.
10
Question 11. Let ABC be an equilateral triangle, and let P stand for an
arbitrary point inside the triangle. Is it true that
P AB − P AC ≥ P BC − P CB ?
[ [ \ \
P AB − P AC = P BC − P CB .
[ [ \ \
We should consider other cases. Let P 0 denote the symmetric point of P with
respect to Ax. The straightline P P 00 intersects AB and AC at M and N, respec-
tively. Choose B 0 that is symmetric point of B with respect to M N. Then
\0
P AB − P[ AC = P AP ,
[
and
\ \ \0
B0P 0.
P BC − P CB = P BP = P\
We will prove that
P
\ AP 0 ≥ P\
B0P 0. (∗)
Indeed, consider the circumscribed circle (O) of the equilateral triangle AM N.
Since
M
\ B0N = M \ BN ≤ M \ BC = M \ AN = 600 ,
11
B 0 is outside (O). Consider the circumscribed circle (O0 ) of the equilateral triangle
AP P 0 . It is easy to see that (O0 ) inside (O), by which B 0 is outside (O0 ). Hence,
P
\ AP 0 ≥ P\ B 0 P 0 . The inequality (*) is proved.
Question 12. Let (O) denote a circle with a chord AB, and let W be the
midpoint of the minor arc AB. Let C stand for an arbitrary point on the major
arc AB. The tangent to the circle (O) at C meets the tangents at A and B at
points X and Y, respectively. The lines W X and W Y meet AB at points N and
M , respectively.
Does the length of segment N M depend on position of C?
Since
1 _ 1 _
ACW
\ =W\AT = AW = W B
2 2
and AW
\ T = CW
\ A, we obtain that ∆AW T, ∆CW A are similar triangles. Then
W A2 = W T × W C.
It is easy to see that W X is the radical axis of A and (Q), thus it passes through
the midpoint N of segment AT . Similarly, W Y passes through the midpoint M
AB
of segment BT . We deduce M N = .
2
Question 13. Let ABC be a triangle. For some d > 0 let P stand for a point
inside the triangle such that
12
Is the following inequality true
|AM | − |P M | ≥ d,
|AM | + |P B| ≥ |AB| + |P M |,
that follows
|AM | − |P M | ≥ |AB| − |P B| ≥ d.
a) Is P divisible by 24?
b) Do there exist m, n ∈ N such that P is not divisible by 7?
Solution. We have
13
It is easy to prove P is divisible by 8, and by 3.
b) It suffices to chose m, n such that the remainders of those divided by 7 are
not 0 and distinct. For instance, m = 2 and n = 1.
Question 15. Let S denote a square of side-length 7, and let eight squares with
side-length 3 be given. Show that it is impossible to cover S by those eight small
squares with the condition: an arbitrary side of those (eight) squares is either
coincided, parallel, or perpendicular to others of S.
14